lawyers in India

Conflict between Interest and Duties of a Lawyer

Written by: Amanjot Malhi - From University School of Law and Legal Studies, Guru Gobind Singh Indraprastha University
Torts Law in India
Legal Service India.com
  • As DINKER in his Legal Ethics observes,
    A lawyer will be constantly confronted with conflicting loyalties which he may have to reconcile. He is answerable not only to his client whose interests it is his primary duty to serve and promote, but also to the Court of which he is an officer and further to his colleagues at the Bar and to the traditions of the Profession.

    Scope of Conflict
    "Conflict is the gadfly of thought. It stirs us to observation and memory. It instigates to invention. Conflict shocks us out of sheep like passivity, and sets us at noting and contriving . . . It is the sine qua non of reflection and ingenuity."

    Whenever interest and duty come into conflict, duty ought to prevail. In the life of an advocate difficult situations do frequently arise; for instance, fill in his adversary in a subsequent suit, the client may not engage you but his opponent must be willing to do so with motives good or bad. The opponent may try to get information from you relating to previous case, which an advocate ought not to do. All the communications made, between client and his lawyer is privileged and such privilege is perpetual. The adversary of your previous client may offer you engagement in a second and third case against a third party and may try to create intimacy with you. In such situations one will rarely err if he keeps in his mind a high sense of honour and conscientious desire to follow right.

    Quoting from In re Jans , Justice Tongue wrote that a cardinal rule of legal ethics says
    "It is never proper for a lawyer to represent clients with conflicting interest no matter how carefully and thoroughly the lawyer discloses the possible effects and obtains consents."

    A lawyer should not appear before any authority of which he is a member in a case against it. A lawyer should not accept a brief in which he has acted in a judicial or quasi-judicial character, e.g., as an Arbitrator. An advocate should not appear in case in which he has given a testimony. This is not on account of conflict between interest and duty, but is based on a different principle; namely, it might tend to throw suspicion on The advocate character which may entail loss of respect for the profession as a whole and diminish public confidence in the purity of administration of justice.

    Our Constitution enshrines that the right to consult and to be defended by a lawyer of one's choice is a fundamental right of a person accused of an offence; and so, it is duty of a lawyer to defend such person. Because of this constitutional injunction, there is absolutely no conflict between interest and duty in criminal cases. However, some difficult may arise in case where the accused confesses his guilt to the lawyer. The answer may be elucidated in the following words:
    He had expressed in England and South African lawyers were consciously or unconsciously let into untruth for the sake of their clients. He vehemently opposed an English lawyer when he advocated that the duty of the lawyer was to defend a client even if he knew that he was guilty. Gandhi on the other hand was emphatic that the duty of a lawyer was to place correct facts before the judge and to help him to arrive at the truth, and not to prove the guilty as innocent.

    In Oceanic Life Ltd v HIH Casualty & General Insurance Ltd , Austin J said the following:
    In the realm of conflicts of interest and conflicts of duty, the lawyer's duty to the court may not be much different from his or her fiduciary duties to former and present clients. However, the duty to the court tends to be expressed in such a way as to emphasise the public interest in preserving confidence in the administration of justice and therefore in the appearance as well as the reality of independence, and the court's practical approach to its supervisory discretions...

    Lord Cozens-Hardy MR in Moody v Cox said that:
    'A man may have a duty on one side and an interest on another. A solicitor who puts himself in that position takes upon himself a grievous responsibility. A solicitor may have a duty on one side and a duty on the other, namely, a duty to his client as solicitor on the one side and a duty to his beneficiaries on the other; but if he chooses to put himself in that position it does not lie in his mouth to say to the client "I have not discharged that which the law says is my duty towards you, my client, because I owe a duty to the beneficiaries on the other side." The answer is that if a solicitor involves himself in that dilemma it is his own fault. He ought before putting himself in that position to inform the client of his conflicting duties, and either obtain from that client an agreement that he should not perform his full duties of disclosure or say-which would be much better-"I cannot accept this business." I think it would be the worst thing to say that a solicitor can escape from the obligations, imposed upon him as solicitor, of disclosure if he can prove that it is not a case of duty on one side and of interest on the other, but a case of duty on both sides and therefore impossible to perform.'

    The thrust of this passage is that if a solicitor puts himself in a position of having two irreconcilable duties it is his own fault. If he has a personal financial interest which conflicts with his duty, he is even more obviously at fault. It was later on quoted in Hilton v. Barker.

    Further the interest of the parties is to seek a favourable decision and their duty is limited to complying with the rules of the court, giving truthful testimony and refraining from taking positive steps to deceive the court but the interest and duty of the advocate is much more complex, because it involves divided loyalties.

    Potential For Conflict
    It was held in the case of Abse and others v Smith and another that
    A lawyer wishes to promote his client's interests and it is his duty to do so by all legitimate means. But he also has an interest in the proper administration of justice to which his profession is dedicated and he owes a duty to the court to assist in ensuring that this is achieved. The potential for conflict between these interests and duties is very considerable yet the public interest in the administration of justice requires that they be resolved in accordance with established professional rules and conventions and that the judges shall be in a position to assume that they are being so resolved. There is thus an overriding public interest in the maintenance amongst advocates not only of a general standard of probity, but of a high professional standard, involving a skilled appreciation of how conflicts of duty are to be resolved.

    Dubin J.A. remarked in Re Regina and Speid , at p. 21:
    We would have thought it axiomatic that no client has a right to retain counsel if that counsel, by accepting the brief, puts himself in a position of having a conflict of interest between his new client and a former one.

    A Word of Caution
    What If a litigant could achieve an undeserved tactical advantage over the opposing party by bringing a disqualification motion or seeking other "ethical" relief using the integrity of the administration of justice or conflict of interest merely as a flag of convenience? Will the fairness of the process would be undermined?
    A similar situation arose in R. v. Parsons, where the accused was charged with the first degree murder of his mother. The Crown sought to remove defence counsel on the basis that he had previously acted for the father of the accused in an unrelated matrimonial matter, and might in future have to cross-examine the father at the son's trial for murder.

    The accused and his father both obtained independent legal advice, after full disclosure of the relevant facts, and waived any conflict. The father also waived solicitor-client privilege. The court was satisfied there was no issue of confidential information. On these facts, the court concluded that "public confidence in the criminal justice system might well be undermined by interfering with the accuser's selection of the counsel of his choice" (Para. 30).

    Advocacy - Is It A Profession or A Business?

    Ultimately, as is so often the case, it is apt to heed the words of Sir Owen Dixon in setting the 'bar': Unless high standards of conduct are maintained by those who pursue a profession requiring great skill begotten of special knowledge, the trust and confidence of the very community that is to be served is lost and thus the function itself of the profession is frustrated.

    It has been repeatedly stressed that the practice of law is not a business. It is a profession in which duty to public service, not money is the primary consideration. Lawyering is not meant to be moneymaking venture, which necessarily reaps profits. Making of livelihood should be secondary consideration of lawyers who must subordinate their interests.

    In Bar Council of Maharashtra v. M. V. Dabholkar , it was stated that:

    Members of bar are not a guild of candle makers or butcher's association. They are a class separate.
    Then there is the theory of Professional Paradigm Shift as given by Russell G. Pierce, which states that the profession should also be treated as a trade as there is no harm to erase the line between profession and trade.

    Soli J. Sorabjee states his views on this point in Lawyers as Professionals . He states that at present the public image of lawyers is far from flattering. They are seen as fortune seekers rather than seeking to serve, a selfish class, who, on account of the special knowledge and expertise, provide services on such terms as they please. In short, the profession of law is regarded as a money making racket. Today people are apt to agree with Dean Swift's description of lawyers as,
    "a society of men bred up from their youth in the art of proving by words multiplied for the purpose, that white is black and black is white according as they are paid." No longer is the profession of law regarded as a noble one.

    He further goes on to say that what are the reasons for this said decline? The main reason is that lawyers, as also other professionals like doctors for example, have forgotten what is entailed in a profession and their proper role in society.

    It is worthwhile turning to dictionaries once in a while. Webster defines profession as "calling requiring specialised knowledge and often long and intensive preparations including instructions in skills and methods... and committing its members, to continue studies and to a kind of a work which has for its prime purpose the rendering of public service." Roscoe Pound summed up the matter with admirable aptness when he said; "historically, there are three ideas involved in a profession; organisation, learning and a sprit of public service. These are essential. The remaining idea, which of gaining of livelihood is incidental."

    It is forgotten that the essential difference between business and a profession is that while the chief end of business is personal gain, the main goal of profession is professional service. Of late lawyers seem to operate on the law of demand and supply and the forces of commercialism have overtaken the profession by and large. The idea that professionals are for the people and the people are not for the professionals sound like a strange and alien doctrine.

    Today the fees charged by some lawyers are staggering. He later says that:
    I was told that a middle rank lawyer charged Rs. 60,000/- for an application for an adjournment in the Bombay high Court. The fees charged by some seniors in the SLP's in the Supreme Court are enormous. Lawyers charge fee even when they have not put in an appearance and it is a disgusting sight to see some lawyers plead with the judge to record their appearance to enable them to collect fees from their absent clients.

    Apparently lawyers have forgotten that they are the guardians of noble ideals and traditions, and not mere traders in the market place, whose sole aim is the amassing of vast fortunes. It is not suggested that lawyers should not charge for their services and charge well in heavy matters where there are corporate clients. A lawyer needs to make money like any other person and it is not expected to live on love and fresh air. But his main purpose and desire should be of rendering service to those who seek his aid and also to the community of which he is a necessary part. To those unable to pay adequately or not at all professional services should be freely and cheerfully given. If doctors can free medical clinics why should lawyers not run free legal clinics atleast once a week? Lawyers must devote some part of their time and services free of charge for the benefit of the poor and oppressed.

    The legal profession has a social dimension. And the lawyer has the social role to play in society. The social dimension becomes a very relevant issue when we look at the manner in which legal services remain by and large inaccessible to the common man, particular in a poor country like us.

    Conflict of Duty – What Are The Conflicting Duties of A Lawyer?

    Imagine you are a client of a lawyer and that you have been so, intermittently, for the last 20 years. The lawyer has prepared your conveyancing deeds and your will. Your certificates of title may be stored in the lawyer's office. The lawyer may have a power of attorney in relation to your personal affairs. You may have been a party to litigation and the lawyer counselled you, dealt with your opponent and appeared on your behalf. The lawyer has and continues to serve your interests faithfully. Now imagine that same lawyer serves a letter of demand on you in relation to a matter in which he or she had formerly acted on your behalf.

    Duty of A Fiduciary Relationship

    Such conduct is generally seen as inappropriate and unethical. Why? The answer resides in the nature of the relationship between lawyer and client. It is intimate -- the client reposes trust and confidence in the lawyer. Indeed, the lawyer is in a fiduciary relationship with the client. More than that, of all the fiduciary relationships known to the law, the lawyer-client relationship is one of the most recognizable. The common law system of justice would not function without it. The public derives, in part, its confidence in the administration of justice from the fidelity of a lawyer to his or her client. It is for this reason that courts have required high standards of propriety from a lawyer.

    In Alexander v Perpetual Trustees WA Ltd , Davies AJA described conflicts of duty as 'insidious thing[s]'. They cloud the mind. Aspects of the lawyer's duty of care, which ought to be seen clearly and distinctly, are seen in a 'hazy light'.

    Lawyers have a fiduciary obligation to avoid 'conflicts of duty'. Conflicts arise when a lawyer who owes a duty to one client undertakes a similar duty towards another client either simultaneously ('present client conflict') or successively ('former client conflict')

    Thomas J said in Kooky Garments Ltd v Charlton:
    As part of their professional responsibility ... lawyers must ensure that they do not appear in a matter in which they have an actual or potential conflict of duty or where, by reason of their relationship with their client, their professional independence could be called in question.

    Duty of Loyalty

    It's relevant here to quote Trial of Queen Caroline (1821), by J. Nightingale, vol. II, The Defence, Part 1, at p. 8 wherein a declaration was made of an advocate's duty of loyalty by Henry Brougham, later Lord Chancellor, in his defence of Queen Caroline against the charge of adultery brought against her by her husband, King George IV. He thus addressed the House of Lords:
    [A]n advocate, in the discharge of his duty, knows but one person in all the world, and that person is his client. To save that client by all means and expedients, and at all hazards and costs to other persons, and, among them, to himself, is his first and only duty; and in performing this duty he must not regard the alarm, the torments, the destruction which he may bring upon others. Separating the duty of a patriot from that of an advocate, he must go on reckless of consequences, though it should be his unhappy fate to involve his country in confusion.

    While the Court is most often preoccupied with uses and abuses of confidential information in cases where it is sought to disqualify a lawyer from further acting in a matter, the duty of loyalty to current clients includes a much broader principle of avoidance of conflicts of interest, in which confidential information may or may not play a role as was quoted in various cases such as Montreal Trust Co. of Canada v. Basinview Village Ltd. ; Enerchem Ship Management Inc. v. Coastal Canada (The) ; Jans v. Coulter (G.H.) Co. ; Stewart v. Canadian Broadcasting Corp.

    Duties of A Lawyer – In Australia, England and Canada

    In Australia, England and Canada, the duties of a lawyer to their client arise both in equity and at common law. At common law, a lawyer's retainer imposes an obligation to be skilful and careful. Failure to fulfill this obligation may lead to liability in contract, or in tort for negligence. In addition, the client must be informed of everything which the lawyer knows will be of assistance to the client in relation to matters within the lawyer's retainer.

    In Spector v Ageda, Megarry J observed:
    [A lawyer] must put at his client's disposal not only his skill but also his knowledge, so far as is relevant, and if he is unwilling to reveal his knowledge to his client, he should not act for him. What he cannot do is to act for the client and at the same time withhold from him any relevant knowledge that he has...

    In equity, the relationship of lawyer and client is recognised as a fiduciary relationship and carries with it obligations to act with absolute fairness and openness towards the client. Gillard J observed in World Medical Manufacturing Corporation v Phillips Ormonde & Fitzpatrick Lawyers (a firm) that:
    As a general proposition the minimum duties which are owed are first, that a fiduciary must not make a personal profit from his position [apart from the fees for his services] and secondly, he must not allow personal interest and duty to conflict. The three grounds on which courts in England, Australia and Canada have restrained lawyers from acting in conflict of duty situations are as follows: -

    A) Duty of Confidentiality

    The Oxford Dictionary of English defines the word 'confidential' as 'intended to be kept secret' and 'entrusted with private or restricted information.' The obligation of confidentiality has long been recognised as a critical feature of the lawyer-client relationship. In Rakusen v Ellis, Munday & Clarke , Fletcher Moulton LJ observed that

    The duty of confidentiality is particularly onerous in relation to lawyers.

    That is because the confidential character of the lawyer-client relationship is 'in the eyes of the law the very highest.'

    Whether founded in contract or equity , the duty to preserve confidentiality is unqualified. It is a duty to keep the information confidential , not merely to take all reasonable steps to do so. Moreover, it is not only a prohibition against communication to third parties; it is a duty not to misuse confidential information without the consent of the client.

    In Drabinsky v. KPMG where the plaintiff sought an injunction restraining the accounting firm KPMG (of which the plaintiff was a client) from further investigating the financial records of a company of which the plaintiff was a senior officer, Ground J., grouping together lawyers and accountants, said, at p. 567:
    I am of the view that the fiduciary relationship between the client and the professional advisor, either a lawyer or an accountant, imposes duties on the fiduciary beyond the duty not to disclose confidential information. It includes a duty of loyalty and good faith and a duty not to act against the interests of the client. [Emphasis added.]

    B) Fiduciary Duty of Loyalty

    Equity imposes upon fiduciaries duties of the utmost good faith and the highest standards, emphasising undivided loyalty and the avoidance of conflicts of interest and/or duty. Lawyers, as a recognised professional class, are in a fiduciary relationship with their clients.

    In Spincode , Brooking JA traced the phrase back to 1928 when Cardozo CJ in Meinhard v Salmon spoke of the rule of undivided loyalty affecting those bound by fiduciary ties. Professor Finn said the following in relation to the effect of the duty:
    Loyalty's effect is two fold. First, if the fiduciary is being remunerated by either or both of the parties, the 'conflict of duty interest' theme in the fiduciary's obligation requires him to disclose to each client that he is being remunerated by the other. Secondly, much more importantly, until each client agrees to the contrary, or unless there is a legally acknowledged custom to the contrary, each client is entitled to, and is entitled to assume that he has, the undivided loyalty of the fiduciary he has engaged. The rule is simple and inexorable: 'Fully informed consent apart, an agent cannot lawfully place himself in a position in which he owes a duty to another which is inconsistent with his duty to his principal.

    Unless a litigant is assured of the undivided loyalty of the lawyer, neither the public nor the litigant will have confidence that the legal system, which may appear to them to be a hostile and hideously complicated environment, is a reliable and trustworthy means of resolving their disputes and controversies as was quoted in R. v. McClure , at Para. 2 and in Smith v. Jones.

    As O'Connor J.A. (now A.C.J.O.) observed in R. v. McCallen, at p. 67:
    ... the relationship of counsel and client requires clients, typically untrained in the law and lacking the skills of advocates, to entrust the management and conduct of their cases to the counsel who act on their behalf. There should be no room for doubt about counsel's loyalty and dedication to the client's case.
    The duty of loyalty is intertwined with the fiduciary nature of the lawyer-client relationship. One of the roots of the word fiduciary is fides, or loyalty, and loyalty is often cited as one of the defining characteristics of a fiduciary as was held in the case of McInerney v. MacDonald and in Hodgkinson v. Simms.

    C) Administration of Justice

    In Davies v Clough , Shadwell V-C said:
    all Courts may exercise an authority over their own officers as to the propriety of their behaviour; for applications have been repeatedly made to restrain [lawyers] who had acted on one side from acting on the other, and those applications have failed or succeeded upon their own particular grounds, but never because the Court had no jurisdiction.

    Hence, in determining whether to disqualify a lawyer, the court may consider whether the continued representation would be subversive to the appearance of loyalty and therefore detrimental to public confidence in the legal system. The concern is that 'justice [should] not only be done but [should] appear to be done'.

    Essentials for Claiming Injunction

    Common to each kind of conflict, whether present or former client conflict, is that the matters which underlie an injunction application must be the same or closely related and that the clients must have adverse interests.

    AA Same or Closely Related Matters
    It has been observed that, regardless of the nature of conflict, the matters which subsist in the alleged conflict must be the 'same or closely related' .

    BB Adverse Interests
    In addition to the above requirement, the parties must also have 'adverse' or 'opposing' interests. This aspect of the law in England and Australia is fairly clear. Both the House of Lords in Prince Jefri Bolkiah and the Court of Appeal in Spincode observed that the parties' interests must be adverse.

    His Lordship in the case of Prince Jefri Bolkiah observed that the case was authority for two propositions:
    (i) that there is no absolute rule of law in England that a lawyer may not act in litigation against a former client; and
    (ii) that the lawyer may be restrained from acting if such a restriction is necessary to avoid a significant risk of the disclosure or misuse of confidential information belonging to the former client.

    His Lordship then observed that the former client bore the onus of proving that the lawyer possessed and continues to possess his or her confidential information and that the lawyer is proposing to act for another client with an adverse interest in a matter to which the information might be relevant. If this was proven, the onus then shifted to the lawyer to prove that effective measures had been taken to ensure that there was no risk of disclosure of the client's confidential information.

    Conflict of Duty - Remedy

    When the remedy for injunction is sought to restrain a lawyer from acting for a party, the principles governing the situation vary depending upon whether the applicant is a present or former client. There are three bases on which a court may grant an injunction:
    # misuse of a client's confidential information;
    # breach of a lawyer's fiduciary duty of loyalty; and
    # the inherent jurisdiction of the court over its officers.

    Nevertheless, for the purposes of an application for an injunction on the grounds of conflict of duty, typically only a client, namely the entity, will have standing to restrain a lawyer. In rare instances, however, the client's officers will be considered as the client.

    In Village Roadshow Ltd v Blake Dawson Waldron, the law firm was acting for Permanent Trustee Co Ltd (a trustee company) in relation to a scheme of company arrangement. By that arrangement, Village Roadshow Ltd was proposing to buy back a certain class of its shares. The law firm, which had acted and continued to act for Permanent Trustee Co Ltd, began to act for certain shareholders of Village Roadshow Ltd who sought to challenge the scheme of company arrangement. Although Village Roadshow Ltd was not a client of the law firm, the court allowed it to bring an application seeking to restrain the law firm from acting on behalf of the shareholders. It appears that the basis for this finding was that the two companies, Permanent Trustee Co Ltd and Village Roadshow Ltd, had similar interests in upholding the scheme of company arrangement and, on that basis, one or the other was entitled to apply to the court seeking to restrain the law firm from acting.

    Test To Determine Conflict of Duty
    In Holborow v Macdonald Rudder, Heenan J helpfully discussed the circumstances in which the jurisdiction of the court should be invoked. His Honour observed:
    If there are circumstances which are likely to imperil the discharge of [the] duties to a court by a [lawyer] acting in a cause, whether because of some prior association with one or more of the parties against whom the lawyer is then to act, or because of some conduct by the lawyer, whether arising from associations with the client or a close interest which gives rise to the fair and reasonable perception that the lawyer may not exercise the necessary independent judgment, a court may conclude that the lawyer should be restrained from acting ...

    Tests for determining if there is conflict of duty was stated in E. S. Reddy v. Chief Secretary, A.P - area of conflict of duty was pointed out.

    Avoidance of Conflict of Duty

    It is relevant here to quote the phrase used by Lord Cozens-Hardy MR in Moody v Cox
    If a house owner contracts to sell his house to one purchaser for £240,000 and then a week later contracts to sell it to another purchaser for £250,000, he assumes two contractual duties which are on the face of it irreconcilable, unless the seller has grounds for rescinding either contract, or can persuade one or other purchaser to release him from his obligation. That is so whether he enters into the second contract with his eyes open, in the hopes of making a larger profit, or whether (rather improbably) he does so inadvertently. It is no answer for him to say to either purchaser: I am sorry, I am obligated to another. His dilemma is his own fault.

    A Fully Informed Consent
    A lawyer may act for different parties notwithstanding that a conflict of duty may arise, provided the lawyer has the informed consent of each of the parties. In Clark Boyce v Mouat, the Privy Council defined 'informed consent' as:
    Consent given in the knowledge that there is a conflict between the parties and that as a result the lawyer may be disabled from disclosing to each party the full knowledge which he possesses as to the transaction or may be disabled from giving advice to one party which conflicts with the interests of the other.

    B Chinese Walls

    The establishment of internal rules and procedures designed to prevent the passage of confidential information from one part of a firm of lawyers to another is often referred to as the erection of a 'Chinese wall'. Generally, this involves the giving of undertakings , the imposition of restraints upon persons and/or limitations upon communications between various persons.

    The term 'Chinese wall', although routinely used in legal vernacular, is an 'imprecise metaphor'. For this reason Straughton LJ in Re a Firm of Solicitors preferred to call it an 'information barrier'.

    Where there has been the previous relationship of solicitors and client and the solicitor at the date of his proposed new retainer possesses relevant confidential information, in the ordinary course the court will in my view grant an injunction restraining the solicitor acting . . . But in the case where without any such previous relationship a party's solicitor illegitimately becomes possessed of confidential information of the other party to the suit or dispute, in the ordinary course the court will merely grant an injunction restraining the solicitor making use of that information; it will not prohibit his continuing to act as was held in English and American Insurance Co Ltd v Herbert Smith and Goddard v Nationwide Building Society.

    Conflict of Interest – What Are The Conflicting Interests of A Lawyer?

    Lawyers have a duty to consider potential conflicts at the outset of an engagement and to decline proffered employment when such conflicts are likely. Even careful conflicting-checking, however, will not eliminate the risk of unforeseeable conflicts arising after the lawyer or firm has commenced multiple representations.

    A conflict of interest is a situation in which someone in a position of trust, such as a lawyer, has competing professional or personal interests. Such competing interests can make it difficult to fulfill his or her duties impartially. A conflict of interest exists even if no unethical or improper act results from it. A conflict of interest can create an appearance of impropriety that can undermine confidence in the person, profession, or court system.

    John S. Dzienkowski defines Positional Conflict of Interests as:
    "A positional conflict of interest occurs when a law firm adopts a legal position for one client seeking a particular legal result that is directly contrary to the position taken on behalf of another present or former client, seeking an opposite legal result, in a completely unrelated matter."

    Black's Law Dictionary
    Black's Law Dictionary defines Conflict of Interests as
    A real or seeming incompatibility between one's private interests and one's public or fiduciary duties. A real or seeming incompatibility between the interests of two of a lawyer's clients, such that the lawyer is disqualified from representing both clients if the dual representation adversely affects either client or if the clients do not consent.

    As early as 1280, a London Ordinance forbade attorneys from representing adverse parties in the same action and from dropping one client to represent another in the same case.

    Conflict of Interest
    Conflict of Interest may be defined as a
    "a situation in which a person, such as a public official, an employee, or a professional, has a private or personal interest sufficient to appear to influence the objective exercise of his or her official duties. "

    In Ramrakha v. Zinner , Harradence J.A., concurring, observed, at Para. 73:
    A solicitor is in a fiduciary relationship to his client and must avoid situations where he has, or potentially may, develop a conflict of interests .... The logic behind this is cogent in that a solicitor must be able to provide his client with complete and undivided loyalty, dedication, full disclosure, and good faith, all of which may be jeopardized if more than one interest is represented.

    New York Code of Professional Responsibility

    Under the Code, a lawyer may not continue the concurrent representation of multiple clients
    "If the exercise of independent professional judgment on behalf of a client will be or is likely to be adversely affected by the lawyer's representation of another client, or if it would be likely to involve the lawyer in representing differing interests, unless the conflict is capable of being, and is, consented to.

    Thrust Upon Conflicts
    "Thrust upon" conflicts are defined as conflicts between two clients that
    (1) did not exist at the time either representation commenced, but arose only during the ongoing representation of both clients, where
    (2) the conflict was not reasonably foreseeable at the outset of the representation,
    (3) the conflict arose through no fault of the lawyer, and
    (4) the conflict is of a type that is capable of being waived but one of the clients will not consent to the dual representation.

    Although the "thrust upon" conflict may be unforeseeable and arise through no fault of the lawyer or law firm affected, when it gives rise to a concurrent conflict, the lawyer must nevertheless take action to avoid any violation of rules. The customary response to such conflicts is for the lawyer to withdraw as necessary to avoid the conflict.

    When client relationships change during the course of a representation, the lawyer should first determine whether the changed circumstances create an actual conflict.

    Scenario 1: A law firm represents Client A in a breach of contract suit against Company B. During the pendency of that suit, Client C, a long-time ongoing client of the law firm, acquires Company B in a stock sale, and Company B becomes a wholly owned subsidiary of Client C. The law firm (which does not represent Client C in the acquisition of B) informs Clients A and C that it wishes to continue to represent each of them in their respective matters. Client A consents to a conflict of interest waiver, but Client C does not. May the law firm continue to represent at least one client, and if so, may the law firm choose which client to represent?

    Scenario 2: A law firm has advised Client A for several years regarding various intellectual property licensing issues. The law firm has also advised Client B for several years on general corporate transactional matters not involving intellectual property licensing, including current negotiations with Company C to form a joint venture. During the course of those negotiations, Client A acquires Company C. Upon learning of the merger, the law firm seeks to obtain conflict of interest waivers from Clients A and B so that it may continue to represent both clients in their respective matters. Client A agrees to provide the necessary conflict of interest waiver, but Client B does not. May the law firm continue to represent at least one of the clients, and if so, may the law firm choose which client to represent?

    In the case of J.P. Morgan Chase Bank v. Liberty Mutual Insurance Co. , it was held that a subsidiary of a corporate client is also a client for conflicts purposes because "the relationship [between the two] is extremely close and interdependent, both financially and in terms of direction;" among other things they operated from the same headquarters, shared the same board of directors, and the general counsel (and senior vice president) of the parent was also the general counsel (and senior vice president) of the subsidiary).

    Under the Code, a lawyer may not take on or continue the concurrent representation of multiple clients if the representation would "involve the lawyer in representing differing interests" or if "the exercise of independent professional judgment in behalf of a client will be or is likely to be adversely affected," unless the lawyer obtains the consent of each client affected by the conflict. It is well settled that this means a lawyer may not oppose a current client in any matter, even if the matter is totally unrelated to the firm's representation of the client, without consent from both clients.

    In the case of Cinema 5, Ltd. v. Cinerama, Inc. , it was held that "where the relationship [with a client] is a continuing one, adverse representation is prima facie improper, and the attorney must be prepared to show, at the very least, that there will be no actual or apparent conflict in loyalties or diminution in the vigour of representation") (internal citation omitted).

    Therefore, ordinarily, when two clients will not consent to a conflict of interest, and the conflict requires consent, the law firm or the lawyer must withdraw from representation of at least one of the clients.

    American Bar Association Model Rules of Professional Conduct

    American Bar Association Model Rules of Professional Conduct (Rule 1.7), have taken this one step further and adopted an express "thrust upon" exception to the general prohibition against simultaneously representing two clients whose interests are directly adverse.

    Model Rule 1.7 states: "(a) Except as provided in paragraph (b), a lawyer shall not represent a client if the representation involves a concurrent conflict of interest. A concurrent conflict of interest exists if: (1) the representation of one client will be directly adverse to another client; or (2) there is a significant risk that the representation of one or more clients will be materially limited by the lawyer's responsibilities to another client, a former client or a third person or by a personal interest of the lawyer. (b) Notwithstanding the existence of a concurrent conflict of interest under paragraph (a), a lawyer may represent a client if: ... (4) each affected client gives informed consent, confirmed in writing."

    It has been adopted by various states of America such as District of Columbia, Delaware.

    This Rule governed the practice of law in the District of Columbia from January 1, 1991, through January 31, 2007. As of February 1, 2007, the New Rules took effect.
    (a) A lawyer shall not advance two or more adverse positions in the same matter.
    (b) Except as permitted by paragraph (c) below, a lawyer shall not represent a client with respect to a matter if:

    (1) That matter involves a specific party or parties and a position to be taken by that client in that matter is adverse to a position taken or to be taken by another client in the same matter even though that client is unrepresented or represented by a different lawyer;

    (2) Such representation will be or is likely to be adversely affected by representation of another client;

    (3) Representation of another client will be or is likely to be adversely affected by such representation;

    (4) The lawyer's professional judgment on behalf of the client will be or reasonably may be adversely affected by the lawyer's responsibilities to or interests in a third party or the lawyer's own financial, business, property, or personal interests.
    (c) A lawyer may represent a client with respect to a matter in the circumstances described in paragraph (b) above if each potentially affected client provides consent to such representation after full disclosure of the existence and nature of the possible conflict and the possible adverse consequences of such representation.
    (d) If a conflict not reasonably foreseeable at the outset of representation arises under paragraph (b)(1) after the representation commences, and is not waived under paragraph (c), a lawyer need not withdraw from any representation unless the conflict also arises under paragraphs (b)(2), (b)(3), or (b)(4).

    Bar Council Rules of Philippines

    Rule 1503 of Bar Council Rules of Philippines states that
    A lawyer shall not represent conflicting interest except by written consent of all concerned given after a full disclosure of the facts.

    Oregon Rules of Professional Conduct

    Oregon Rules of Professional Conduct sections 1.7-1.11 retain, for the most part, the salient provisions of former DR 5-105 which are: -
    a) A lawyer shall decline proffered employment if the exercise of his independent professional judgment in behalf of a client will be or is likely to be adversely affected by the acceptance of the proffered employment, except to the extent permitted under DR5-105(C).

    b) A lawyer shall not continue multiple employments if the exercise of his independent professional judgment in behalf of a client will be or is likely to be adversely affected by his representation of another client, except to the extent permitted under DR-505(C).

    c) In the situations covered by DR 5-105(A) and (B), a lawyer may represent multiple clients if it is obvious that he can adequately represent the interest of each and if each consents to the representation after full disclosure of the possible effect of such representation on the exercise of his independent professional judgment on behalf of each.

    Ohio Model Rules of Professional Conduct

    Ohio Model Rules of Professional Conduct R. 1.7 (2004) . The full text for the conflict of interest for current clients rule is as follows:
    (a) Except as provided in paragraph (b), a lawyer shall not represent a client if the representation involves a concurrent conflict of interest. A concurrent conflict of interest exists if:
    a. the representation of one client will be directly adverse to another client; or
    b. there is a significant risk that the representation of one or more clients will be materially limited by the lawyer's responsibilities to another client, a former client or a third person or by a personal interest of the lawyer.

    (b) Notwithstanding the existence of a concurrent conflict of interest under paragraph (a), a lawyer may represent a client if:
    a. the lawyer reasonably believes that the lawyer will be able to provide competent and diligent representation to each affected client;
    b. the representation is not prohibited by law;
    c. the representation does not involve the assertion of a claim by one client against another client represented by the lawyer in the same litigation or other proceeding before a tribunal; and (4) each affected client gives informed consent, confirmed in writing.
    as

    Code of Conduct for European Lawyers

    The Code of Conduct for European Lawyers (Rule 3.2) . The full text for the conflict of interest for current clients rule is as follows:
    3.2.1. A lawyer may not advise, represent or act on behalf of two or more clients in the same matter if there is a conflict, or a significant risk of a conflict, between the interests of those clients.
    3.2.2. A lawyer must cease to act for both or all of the clients concerned when a conflict of interests arises between those clients and also whenever there is a risk of a breach of confidence or where the lawyer's independence may be impaired.
    3.2.3. A lawyer must also refrain from acting for a new client if there is a risk of breach of a confidence entrusted to the lawyer by a former client or if the knowledge which the lawyer possesses of the affairs of the former client would give an undue advantage to the new client.
    3.2.4. Where lawyers are practising in association, paragraphs 3.2.1 to 3.2.3 above shall apply to the association and all its members.
    The provisions of Article 3.2.1 do not prevent a lawyer acting for two or more clients in the same matter provided that their interests are not in fact in conflict and that there is no significant risk of such a conflict arising.

    Where a lawyer is already acting for two or more clients in this way and subsequently there arises a conflict of interests between those clients or a risk of a breach of confidence or other circumstances where the lawyer's independence may be impaired, then the lawyer must cease to act for both or all of them.
    There may, however, be circumstances in which differences arise between two or more clients for whom the same lawyer is acting where it may be appropriate for the lawyer to attempt to act as a mediator. It is for the lawyer in such cases to use his or her own judgement on whether or not there is such a conflict of interest between them as to require the lawyer to cease to act. If not, the lawyer may consider whether it would be appropriate to explain the position to the clients, obtain their agreement and attempt to act as mediator to resolve the difference between them, and only if this attempt to mediate should fail, to cease to act for them.

    Article 3.2.4 applies the foregoing provisions of Article 3 to lawyers practising in association. For example a firm of lawyers should cease to act when there is a conflict of interest between two clients of the firm, even if different lawyers in the firm are acting for each client. On the other hand, exceptionally, in the chambers form of association used by English barristers, where each lawyer acts for clients individually, it is possible for different lawyers in the association to act for clients with opposing interests.

    Code of Conduct For The Bar of Ireland

    Code of Conduct for the bar of Ireland . It has specific provisions regarding conflict of interests and duty and they are as follows: -
    2.1 In order to perform their functions with due independence and in a manner which is consistent with their duty to participate in the administration of justice a barrister is excluded from occupations which conflict with the duties contained in the Code of Conduct.

    3.1 If by reason of the negligence of his or her instructing solicitor or otherwise a barrister forms the view that there is a conflict of interest between the client and the instructing solicitor, the barrister should advise that it would be in the client's interest to instruct another solicitor. A barrister should use discretion as to whether such advice should be given either at a consultation at which the client and the solicitor are present or be given in writing to the client through the solicitor or given to the solicitor.

    10.1 Barristers may appear for more than one defendant in a criminal trial provided they have satisfied themselves that there is no conflict of interest.

    Law Council of Australia - Model Rules of Professional Conduct And Practice

    Model Rules of Professional Conduct and Practice (Rule 9) the main text of the rule is as follows: -
    Acting for more than one party
    8.2 A practitioner must avoid conflict of interest between two or more clients of the practitioner or of the practitioner's firm.
    Avoiding Conflict of Interest (where practitioner's own interest involved)
    9.1 A practitioner must not, in any dealings with a client:
    9.1.1 allow an interest of the practitioner or an associate of the practitioner to conflict with the client's interest;
    9.1.2 exercise any undue influence intended to dispose the client to benefit the practitioner in excess of the practitioner's fair remuneration for the legal services provided to the client.
    9.2 A practitioner must not accept instructions to act or continue to act for a person in any matter when the practitioner is, or becomes, aware that the person's interest in the matter is, or would be, in conflict with the practitioner's own interest or the interest of an associate.

    Test For Determining Conflict of Interest

    The court then in the case of In re Brandsness articulated this test to determine whether a closed-file conflict of interest exists. A three-factor test can be used to determine if a conflict exists. When the following factors co-exist, a conflict results:

    a) The adverse party is one with whom the accused had a lawyer-client relationship;
    b) The representation of the present client puts the accused in a position adverse to the former client; and
    c) The present matter is significantly related to a matter in which the accused represented the former client.

    In R. v. Chen (2001) , it was stated that the conflict should, of course, be raised at the earliest practicable stage. If the trial is concluded, the conflict of interest may still be raised at the appellate level as a ground to set aside the trial judgment, but the test is more onerous because it is no longer a matter of taking protective steps but of asking for the reversal of a court judgment.

    In R. v. Graff , the Alberta Court of Appeal held that in a post-conviction situation, if an accused is to challenge a conviction or sentence on appeal, he or she must show more than a possibility of conflict of interest; while actual prejudice need not be shown, the appellant must demonstrate the conflict of interest and that the conflict adversely affected the lawyer's performance on behalf of the appellant.

    Conflict Between Interest And Duty – Statutory Analysis
    Duty To The Court
    RULE - 1.
    An advocate shall, during the presentation of his case and while otherwise acting before a court, conduct himself with dignity and self-respect. He shall not be servile and whenever there is proper ground for serious complaint against a judicial officer, it shall be his right and duty to submit his grievance to proper authorities.

    RULE – 4.
    An advocate shall use his best efforts to restrain and prevent his client from resorting to sharp or unfair practices or from doing anything in relation to the court, opposing counsel or parties which the advocates himself ought not to do. An advocate shall refuse to represent the client who persists in such improper conduct. He shall not consider himself a mere mouth-piece of the client, and shall exercise his own judgment in the use of restrained language in correspondence, avoiding scurrilous attacks in pleadings, and using intemperate language during arguments in court.

    Duty To Client
    RULE – 22.
    An advocate shall not, directly or indirectly, bid for or purchase, either in his own name or in any other name, for his own benefit or for the benefit of any other person, any property sold in the execution of a decree or order in any suit, appeal or other proceeding in which he was in any way professionally engaged. This prohibition, however, does not prevent an advocate from bidding for or purchasing for his client any property, which his client may, himself legally bid for or purchase, provided the Advocate is expressly authorized in writing in this behalf.

    Duty To Colleagues
    RULE – 33.
    An advocate who has, at any time, advised in connection with the institution of a suit, appeal or other matter or has drawn pleadings, or acted for a party, shall not act, appear or plead for the opposite party.

    RULE – 36.
    An advocate shall not solicit work or advertise, either directly or indirectly, whether by circulars, advertisements, touts, personal communications, interviews not warranted by personal relations, furnishing or inspiring newspaper comments or producing his photographs to be published in connection with cases in which he has been engaged or concerned. His signboard or nameplate should be of a reasonable size. The sign-board or name-plate or stationery should not indicate that he is or has been President or Member of a Bar Council or of any Association or that he has been associated with any person or organization or with any particular cause or matter or that he specialises in any particular type of worker or that he has been a Judge or an Advocate General.

    RULE - 43.
    An Advocate who has been convicted of an offence mentioned under Section 24A of the Advocates Act or has been declared insolvent or has taken full time service or part time service or engages in business or any avocation inconsistent with his practising as an advocate or has incurred any disqualification mentioned in the advocates Act or the rules made there under, shall send a declaration to that effect to the respective State Bar Council in which the advocate is enrolled, within ninety days from the date of such disqualification. If the advocate does not file the said declaration or fails to show sufficient cause for not filing such declaration provided therefore, the Committee constituted by the State Bar Council under rule 42 may pass orders suspending the right of the advocate to practice.

    Conflict Between Interest And Duty – Judicial Response

    Representing Conflicting Interest
    A. case of conflict is where a solicitor acts for both parties to a transaction without disclosing this to one of them or where having disclosed it he fails, unbeknown to one party, to disclose to that party material facts relative to the other party of which he is aware.

    English Law
    In the case of Clark Boyce v Mouat it was held that:
    There was no general rule of law that a solicitor should never act for both parties in a transaction where their interests might conflict. Instead, a solicitor was entitled to act for both parties in a transaction even where their interests might conflict provided he obtained the informed consent of both parties to his acting. Informed consent in that context meant consent given in the knowledge that there was a conflict between the parties and that as a result the solicitor might be disabled from disclosing to each party the full knowledge which he possessed as to the transaction or might be disabled from giving advice to one party which conflicted with the interests of the other, and if the parties were content to proceed on that basis the solicitor could properly act for both parties. In determining whether a solicitor had obtained informed consent to acting for parties with conflicting interests it was essential to determine precisely what services were required of him by the parties since, if a client in full command of his faculties and apparently aware of what he was doing sought the assistance of a solicitor in the carrying out of a particular transaction, the solicitor was under no duty, whether before or after accepting instructions, to go beyond those instructions by proffering unsought advice on the wisdom of the transaction.

    There is no general rule of law to the effect that a solicitor should never act for both parties in a transaction where their interests may conflict. Rather is the position that he may act provided that he has obtained the informed consent of both to his acting. Informed consent means consent given in the knowledge that there is a conflict between the parties and that as a result the solicitor may be disabled from disclosing to each party the full knowledge which he possesses as to the transaction or may be disabled from giving advice to one party which conflicts with the interests of the other. If the parties are content to proceed upon this basis the solicitor may properly act.

    In Boulting v Association of Cinematograph Television and Allied Technicians Upjohn LJ said:
    . . . the client is entitled to the services of his solicitor who may not charge more than he is legally entitled to and must not put himself into a position where he may owe conflicting duties to different clients. But the person entitled to the benefit of the rule may relax it, provided he is of full age and sui juris and fully understands not only what he is doing but also what his legal rights are and that he is in part surrendering them.

    Farrington v Row McBride & Partners concerned a solicitor who advised a client to invest money in a company, which was also a client of his without disclosing that fact to the potential investor. Richardson J said (at 90):

    A solicitor's loyalty to his client must be undivided. He cannot properly discharge his duties to one whose interests are in opposition to those of another client. If there is a conflict in his responsibilities to one or both he must ensure that he fully discloses the material facts to both clients and obtains their informed consent to his so acting: No agent who has accepted an employment from one principal can in law accept an engagement inconsistent with his duty to the first principal from a second principal, unless he makes the fullest disclosure to each principal of his interest, and obtains the consent of each principal to the double employment. And there will be some circumstances in which it is impossible, notwithstanding such disclosure, for any solicitor to act fairly and adequately for both.

    In the case of Christie v Wilson and others , plaintiff applied to the court to bar P from so acting on the ground that there was a conflict of interest on the part of P and that he would be in breach of Rule 4.1 [a] of the Law Society's Code for Advocacy 1993, which provided that:
    advocates were obliged not to accept any brief if to do so would cause them to be professionally embarrassed and that advocates would be so professionally embarrassed '(e) if they have been responsible for deciding on a course of action and the legality of that action is in dispute in the proceedings.

    The court held that it was a very different thing for a lawyer to advise his client that he might properly do something from taking that decision himself. In the instant case, P had not gone beyond the advisory stage since although he had advised the editor that the article was not in his opinion defamatory, the decision to publish had been that of the editor. It followed that the judge had been incorrect to hold that P was in breach of his professional duty in accepting instructions to act for the second defendant. Accordingly, the appeal was allowed.

    Another important case is of Hilton v Barker Booth and Eastwood (a firm) . In this case the defendant firm of solicitors acted for B in criminal proceedings which resulted in his imprisonment for offences of participating in the management of a company while an undischarged bankrupt, fraudulent trading and obtaining credit while an undischarged bankrupt. A few months after his release, B contacted the claimant, a small-scale property developer, with a proposal for a development. The developer had never previously met B and was unaware of his convictions.

    Subsequently, several meetings took place at the firm's offices between the developer, B and a partner in the firm. The partner knew of B's bankruptcy and his prison sentence, but the developer remained ignorant of them.

    The developer eventually agreed to buy certain land that had been found by B, develop it by the erection of number of flats and then sell the developed property to B. Unbeknown to the developer, B agreed to sell on the flats to a sub-purchaser. The three contracts were exchanged on the same day with the firm acting for both the developer and B. The developer required a deposit from B, which, again unbeknown to the developer, was advanced to B by the firm. B failed to complete, and the transaction proved disastrous to the developer who subsequently brought proceedings against the firm for damages for breach of contract.

    At trial, the judge concluded that the developer would have had nothing to do with the transaction if he had been informed of B's antecedents; that the firm had been in breach of its professional duty in acting for both B and the developer; that the firm could not have informed the developer of B's bankruptcy and convictions without breaching its professional duty to B; that the firm's breach of duty to the developer lay in continuing to act, not in failing to pass on the information; that the developer was therefore entitled to be placed in the position he would have been in if he had instructed an independent solicitor; that the claim had not been advanced on the basis that such a solicitor would have been aware of B's convictions or would have advised the developer to have a credit report; and that accordingly the firm's breach of duty had caused no loss to the developer.

    On that basis, the judge dismissed the claim. His decision was affirmed by the Court of Appeal, which held that the contract between the developer and the firm contained an implied term excusing the firm from disclosing to the developer information that they were legally obliged to someone else to treat as confidential. On the developer's appeal to the House of Lords, their Lordships considered the position of a solicitor who had assumed irreconcilable duties to different clients and whether the firm had, on the facts, placed itself in such a position.

    The Court allowed the appeal and directed that the quantum of damages (if not agreed) should be assessed by a judge.

    In Marks and Spencer plc v Freshfields Bruckhaus Deringer, The defendant firm of solicitors acted for the claimant retailer in the majority of its complex and/or high end contentious work and advised in relation to a number of commercial and employment matters. As a result it had acquired confidential information about the claimant's supply chain and logistical arrangements. The defendant was advising the claimant in connection with negotiations in relation to one of its main contractual arrangements (the main contract) with a view to restructuring it.

    The claimant learned that the defendant was acting on behalf of G in relation to a possible bid for the claimant by a consortium of G's family interests and financial institutions and applied for an injunction prohibiting the defendant from acting for or advising or otherwise assisting G or any related member of the consortium in relation to the acquisition of its shares, assets or business.

    The application was made on the bases:
    (i) that in relation to the main contract there was an actual potential conflict of interest between the interests of the claimant and the interests of the consortium to which the claimant had not consented; and

    (ii) that the defendant possessed confidential information belonging to the claimant that was or could be relevant to the retainer which the defendant had for the consortium. The defendant contended, inter alia, that a conflict of interest required that interests conflict in a single transaction, and that internal barriers and undertakings would suffice to protect confidential information.

    The Court held that:
    The rule that a fiduciary could not act for two principals with potentially conflicting interests without the informed consent of both was not limited to the context of conflicting interests in the same transaction, although there did have to be some reasonable relationship between the two matters. In the instant case there was a real or serious risk of conflict; the main contract was a very important part of the claimant's business and of the tactics of the bid and if the defendants acted for the consortium they would be in direct conflict with their existing duty to act in the best interests of the claimant in connection with the restructuring of the main contract. Moreover, given the great amount of confidential information relating to the affairs of the claimant held within the defendants, effective internal barriers could not be put in place. The injunction would therefore be granted.

    In Prince Jefri Bolkiah v KPMG (a firm) , Lord Millett expanded on these matters in a case, which did not in fact involve solicitors, but accountants who had got into a situation where they were in possession of information from a previous retainer. He said:
    ... a fiduciary cannot act at the same time both for and against the same client, and his firm is in no better position. A man cannot without the consent of both clients act for one client while his partner is acting for another in the opposite interest. His disqualification has nothing to do with the confidentiality of client information. It is based on the inescapable conflict of interest which is inherent in the situation.

    Another case is of Re Schuppan (a bankrupt) wherein the petitioning creditor obtained judgment against the debtor in a contested action in which the debtor was found guilty of fraud and dishonesty. The debtor was later adjudicated bankrupt following his failure to satisfy the judgment. Certain matters in connection with the main action remained outstanding, including an application for a wasted costs order, an inquiry as to damages in associated proceedings in the Isle of Man and an action for slander brought by the debtor against the petitioning creditor's solicitors. Nevertheless, the district judge authorised the trustee in bankruptcy to retain the petitioning creditor's solicitors to advise and assist him in the administration of the debtor's estate, subject to an exception requiring another firm to be retained in relation to the unfinished business. The debtor objected to the trustee in bankruptcy being advised by the petitioning creditor's solicitors on the grounds that a conflict of interest existed and applied to set aside the district judge's authorisation. The application was dismissed and the debtor appealed.
    The Court held that the petitioning creditor was the largest creditor and no difficulties were expected in quantifying the provable debts, the risk of a conflict of interest would appear to be only a distant possibility and thus the appeal would accordingly be dismissed.

    Another case is of Re a firm of solicitors wherein between 1982 and 1985 the appellants a large City of London firm of solicitors with over 100 partners (the firm), acting for a company, ASM, which had taken over the management of two Lloyd's underwriting syndicates, in an investigation into the former management of the two syndicates.

    The investigation attracted wide public interest and was much discussed in the legal profession and in the insurance world. While acting for ASM in the investigation the firm received confidential information from companies (the A & A companies) which were wholly independent of but closely associated with ASM. It was arranged between the two that A & A and its former managing agents would co-operate closely with ASM and the firm and would provide them with full information even if such information could be used against them and, although the A & A companies were not themselves strictly clients of the firm, their relationship with ASM and the firm in the course of the investigation was such that it was proper for the firm to regard the A & A companies as if they were also clients of the firm.

    Accordingly, the firm owed the A & A companies a duty not to communicate confidential information which they had supplied to the firm. As a result of the investigation claims were in due course brought against the former managing agents and an A & A broking subsidiary. All claims were settled in late 1985 or early 1986. Thereafter the firm ceased to act for ASM. In November 1984 one of the A & A companies, SD, commenced an action against the defendant representing himself and the other members of a third Lloyd's syndicate, claiming sums due under quota share reinsurances and for breach of contract.
    The issues raised in the action were closely bound up with the firm's investigation for ASM into the management of the other two syndicates. In November 1990 the defendant instructed the firm to act for him in the action in place of his existing solicitors. The A & A companies, including SD, sought an injunction to restrain the firm from acting for the defendant on the ground that the firm had acquired confidential information and knowledge from the A & A companies when acting for ASM in the investigation of the other two syndicates which was relevant to the litigation between SD and the defendant and that therefore there was a conflict of interest.

    The judge granted the injunction. The firm appealed, contending that they should be prevented from acting for the defendant only if it was probable that SD would be prejudiced thereby and that measures which it proposed to take by erecting a so-called 'Chinese wall' would prevent any leakage of the confidential information it had acquired from the A & A companies in the ASM investigation. The proposed measures consisted of putting all the documents relating to the ASM investigation into special storage, providing a different team to act for the defendant from the team which had acted in the ASM investigation, locating the defendant's team in a different department and a different building from the ASM team, and taking steps to ensure that the two teams did not discuss or communicate with each other about either case.

    The Court held the following:
    (1) There was no general rule that a firm of solicitors who had acted for a former client could never thereafter act for another client against the former client, but a firm of solicitors would not be permitted to act for an existing client against a former client if a reasonable man with a knowledge of the facts would reasonably anticipate that there was a danger that information gained while acting for the former client would be used against him or (per Staughton LJ) there was some degree of likelihood of mischief, i.e. of the confidential information imparted by the former client being used for the benefit of the new client. If (Staughton LJ dissenting) there was such a conflict of interest it was only in very special cases that the court would consider that a Chinese wall would provide an impregnable barrier against the leakage of confidential information;

    (2) (Staughton LJ dissenting) On the facts, a reasonable man with knowledge of all the facts including the measures for a Chinese wall proposed to be taken by the firm would, notwithstanding those measures, still consider that if the firm was allowed to continue to act for the defendant there would be a risk that some of the confidential information provided by the A & A companies to the firm when it was acting for ASM might inadvertently be revealed to the firm's team who were to act for the defendant. Accordingly, the appeal would be dismissed and the injunction restraining the firm from acting for the defendant continued.

    Reason For The Conflict of Interest
    The relationship between a solicitor and his client is one in which the client reposes trust and confidence in the solicitor. It is a fiduciary relationship. But not every breach of duty by a fiduciary is a breach of fiduciary duty. If a solicitor is careless in investigating a title or drafting a lease, he may be liable to pay damages for breach of his professional duty, but that is not a breach of a fiduciary duty of loyalty; it is simply the breach of a duty of care as was said by Millett LJ in Bristol and West Building Society v Mothew (t/a Stapley & Co).

    A solicitor's duty of single-minded loyalty to his client's interest, and his duty to respect his client's confidences, does have their roots in the fiduciary nature of the solicitor-client relationship. But they may have to be moulded and informed by the terms of the contractual relationship as was said by Mason J in Hospital Products Ltd v United States Surgical Corp. The solicitor's duty of single-minded loyalty to his client very frequently makes it professionally improper and a breach of his duty to act for two clients with conflicting interests in the transaction in hand.

    Indian Case Laws
    In the absence of any specific provision regarding conflict between interest and duties of a lawyer, the subject can only be discussed by judicial response.

    Change of Sides
    In the case of H.V. Panchaksharappa v. K.G. Eshwar it was stated that
    Before parting with this appeal, we would, however, like to observe that respondent ought to have, according to the best traditions of the Bar, disclosed to the appellant that he had been retained by the defendant in O.S. No. 119/1986. There may not have been any clash of interest but since the defendant in O.S. No. 237/1986 was his client as a plaintiff in O.S. No. 119/1986, the information should have been disclosed to the appellant. It is just plain and simple obligation of a counsel to disclose such facts to his client. Though, withholding of the information may not amount to professional misconduct, nonetheless the action does not speak well of the conduct of the respondent. On this account we leave the parties to bear their own costs in this appeal.

    Speaking generally it is quite clear that a professional gentleman should as far as possible stick to the side who first engaged him. It might be a very good practice if when gentlemen were offered instructions in any connected case, that they should at least in the first place inform their first client.

    In Emperor v. Rajni Kanta Ghose, A division bench of the Calcutta High Court held that a legal practitioner appearing on both sides was guilty of professional misconduct. The Court further held that:
    As this has happened once and may happen again, it is desirable to state the well-settled general rule that a legal practitioner cannot represent conflicting interests or undertake the discharge of inconsistent duties. when he has once been retained and received the confidence of a client, he cannot accept a retainer from or enter the service of those whose interests are adverse to his client in the same controversy or in a matter so closely allied thereto as to be in effect a part thereof.

    The rule is rigid and is designed not only to prevent the dishonest practitioner from fraudulent conduct, but as well to preclude the honest practitioner from putting himself in a position where he may be required to choose between conflicting interests.

    The full Bench of the Lahore High Court in In the matter of Ramalal Anand held that change of sides as such by counsel is not forbidden by law; change of sides is forbidden if there are confidential communications from one side which may be made use of when the lawyer represents the other party. It is forbidden if the lawyer obtains his own discharge and obtains his own discharge and acts for the opposite party; it is also forbidden if the lawyer accepts a retainer from the opposite party without first offering his services to his original client.

    Principle
    The principal underlying the ban, prohibiting an Advocate from appearing from the opposite party against his former client is that there is the likely hood of conflict of interest between the duties and interest of the Lawyer and also possibility of misuse of the instructions given to him by his former client. It is not a question whether the misuse has been actually made; the mere possibility of such misuse in a matter, which is connected with the previous litigation, is sufficient,

    In order to prevent an Advocate from appearing for the opposite party, what one has to see is whether in the circumstances of a particular, having regard to the steps taken in the litigation of criminal proceedings, it can be reasonably inferred that confidential information could have been imparted, In the case of Earl Cholmondeley v. Lord Clinton. The Lord Chancellor said:

    If there is any ground for any application, either as a motion in the cause or upon the general jurisdiction , it must be furnished by a general principal, not the particular circumstances of the case; otherwise the court must try every such case on its particular circumstances and it cannot be discussed without a disclosure from the solicitor of all he knows.

    It has been reiterated in Halsbury's Laws of England
    A barrister must decline to accept instructions which would render it difficult to maintain his professional independence or would embarrass him. He ought not to appear for two clients whose interest may conflict, or if he is in possession of confidential information relating to the opposite party, or if he is a witness to a material fact.

    In Tajendra Chandra v. Tajendra Lal, a special bench of the Rangoon High Court observed the following: -
    it is clear that an advocate or pleader who has appeared on behalf of one party in a suit ought not to allow himself to be placed in the position in which there might become suspicion, whether well or ill founded that his knowledge of his client's case would be used by him on a subsequent occasion in appearing for another party and against his own client.

    It is settled general rule that a legal practitioner cannot represent conflicting interests or undertake the discharge of inconsistent duties. When he has once been retained and has received the confidence of a client, he cannot accept a retainer from or enter the service of those whose interests are adverse to his client in the same controversy or in a matter so closely allied thereto as to be in effect a part thereof. The rule is right and is designed not only to prevent the dishonest practitioner from fraudulent conduct, but as well to preclude the honest practitioner from putting himself in a position where he may be required to choose between conflicting duties or be led to an attempt to reconcile conflicting interests, rather than to enforce to their full extent the rights or interest which he should alone represent as was held in the case of Day v. Ponsonby.

    But where a legal practitioner did not actually appear for both sides but merely attempted to make the best of the bargain by issuing improper notice to his former client threatening appearance on behalf of the main party knowing fully well that he had rendered himself incapable of accepting brief, a lenient view was taken though the court held that the practitioner was guilty of grossly improper conduct which was not in consonance with the dignity of the profession as was held in the case of Ram Bharosa Kular Bhandari Kalai v. Surndra Nath Thakur.

    A counsel cannot represent clients with conflicting interests. Thus it has been held that counsel cannot represent two different creditors whose interests are know to conflict as was held in the case of The Government Pleader, High Court, Bombay v. Bhagabhai Dayalbhai

    The Patna High Court in Emperor v. Bir Kishore Rai held that the conduct of a pleader in acting for both sides in the same case is grossly improper conduct. The said case was under the Legal Practitioners Act.

    The Allahabad High Court in the case of Saharanpur Grain Chamber Limited v. Maharaj Singh held that when an advocate was consulted by one party is perfectly free to accept the brief against him, if he has not received any information of a confidential nature which will be of use against the party in litigation.
    The law is not that once an advocate is consulted by one party, he may not accept the brief for the other party, no matter what may have been the nature of information conveyed to him during the course of consultation. No doubt the litigants are entitled to protection against unscrupulous members of the legal members of the legal profession. But the members of the legal profession are equally entitled to protection against unscrupulous litigations and if the law were that once a counsel was consulted by one party, he could not under nay circumstances accept a brief against him, the position of the legal profession will be perilous indeed. The onus of proving that confidential information was conveyed lies heavily upon the applicant.

    In another case of Emperor v. Shanti Narayan Manocha , where the pleader drafted the plaint gratuitously as friend and no confidential information was imparted by the plaintiff, the pleader subsequently accepted brief of the defendant. The pleader was not held guilty of misconduct.

    An advocate accepted the brief for the plaintiff and appeared at the initial stage. Subsequently he reported no instructions and then appeared as Govt. Pleader on behalf of State which was added as a defendant, in the suit. The advocate's conduct amounts to misconduct in the case of In the matter of Sri Gurubasappa

    American Case
    Attempting to act for one party to a litigation or business dealings after having engaged to represent the adverse party, and seeking to use the knowledge and secrets acquired from the first for the benefit of the second and the injury of the first, is such culpable conduct as will always justify suspension or disbarment unless satisfactory explanation can be made.

    In the case of Hira Devi v. Digvijay Singh Legal Practitioner's Act, 1876 – pleader acting for one party in litigation should not act for the other party in another litigation arising out of the same dispute. If the lawyer represents both the parties in the same dispute, then this is against the honour of the profession.

    Purchasing Disputed Property From The Client

    In the case of P. D. Gupta v. Rammurti , the advocate purchased from his client his property, which was the subject matter of dispute pending in the Court, thereby creating a conflict between interest and duty [Rule 22]. The advocate was held guilty of professional misconduct and a sentence of debarring him from practice for a period of one year was affirmed.

    Another instance of conflict between interest and duty was in M, an Advocate , where an advocate persuaded his client to execute a power of attorney in favour of a junior who in turn sold the property to the father of the advocate. The junior advocate admitted having done so under the misguidance of his senior. Both were held guilty of professional misconduct.

    It is unprofessional conduct on the art of a legal practitioner to purchase an actionable claim. An actionable claim does not cease to be served, simply because a suit has been instituted in respect of it. It is for the purchasing practitioner to show that such a purchase does not amount to an unprofessional conduct and conflict of interest. In an Oudh case, a practitioner purchased certain property which was to be the subject matter of the litigation in the name of the third person. He got the suit instituted in the name of the third person and he not only appeared in and conducted the case but also took his fee which he got taxed against defendants in spite of the fact that he himself was the real plaintiff. It was held that he was guilty of professional misconduct in the case of Sheo Narayan Lal v. Mir Amjad Ali.

    The matter is expressed very clearly by Lord Chancellor St. Leonards in Lewis v. Hillman
    I should lay it down as a rule that ought never to be departed from that if an attorney or agent can show he is entitled to purchase, yet if instead of openly purchasing he purchases in the name of a trustee or agent without disclosing the fact, no such purchase as that can stand for a single moment.

    And the same doctrine is repeated in the latter case of McPherson v. Watt, where at p. 266 Lord O'Hagan lays down the conditions which must exist to validate a purchase by an attorney from his client, and adds:
    And although all these conditions have been fulfilled, though there has been the fullest information, the most disinterested counsel and the fairest price be made covertly in the name of another without communication of the fact to the pleader the law condemns and invalidate it utterly. There must be uberrima fides between the attorney and the client and no conflict of duty and interest can be allowed to exist.

    Soliciting Briefs
    Another conflict between interest and duty arises under Rule 36 of Bar Council of India Rules wherein it is prescribed that advocate shall not solicit brief or advertise. In the case of In Re, Mr. A an Advocate , that if the advocate knew that it is highly improper to solicit a brief and even then he wrote a post card to the Government of Maharashtra soliciting their briefs, he is very unworthy member of a learned profession.

    Advocate Handing Over Brief To Another Without Client's Consent
    A lawyer when entrusted with a brief is expected to follow the norms of professional ethics and try to protect the interests of his clients, in relation to whom he occupies a position of trust. It is not in accordance with professional etiquette for an advocate to hand over his brief to another to take his place at a hearing (either for the whole or part of the hearing) and conduct the case as if the latter had himself being briefed, unless the client consents to this course being taken.
    It was held in V. C. Rangaburai v. D. Gopalan, it was held that
    Counsel's paramount duty is to the client: accordingly where he forms an opinion that a conflict of interest exists, his duty is to advise the client that he should engage some other lawyer. It is unprofessional to represent conflicting interests, except be express consent given by all concerned after a full disclosure of the facts.

    Taking Money From The Client To Bribe Another

    The Preamble to Chapter II of Part VI of the Rules of the Bar Council of India lays down that an advocate shall at all times comport himself in a manner befitting his status as an officer of the Court, privileged member of the community and a gentleman. Rule 4 of this Chapter provides that an advocate shall use his best efforts to restrain and prevent his client from resorting to sharp and unfair trade practices etc. Now a conflict ensues when a lawyer fails to perform his duty due to certain interests. In the case of Chandra Shekhar Soni v. Bar Council of Rajasthan , the appellant had procured the brief of the complainant in another case on a fee of Rs. 300/- on the representation that he would secure a favourable report from the Radiologist showing that there was a fracture of the skull. Therefore conflict arose because of this monetary interest. It was held that: -
    Nothing should be done by any member of the legal fraternity which might tend to lessen in any degree the confidence of the public in the fidelity, honesty and integrity of the profession. The State Bar Council gave the appellant the benefit of doubt on the first charge that he changed sides in a criminal case, holding that though such conduct on his part was unprofessional, it was not tantamount to professional misconduct. The Disciplinary Committee of the Bar Council of India rightly observes that it failed to appreciate the distinction drawn by the Slate Bar Council as his act in accepting the brief for the accused after having appeared for the complainant was clearly contrary to r. 33 of the Bar Council of India Rules, 1975. We concur with the Disciplinary Committee. It is not in accordance with professional etiquette for an advocate while retained by one party to accept the brief of the other. It is unprofessional to represent conflicting interests except by express consent given by all concerned after a full disclosure of the facts. The appellant would not have appeared for the other side except with the permission of the learned Magistrate. Counsel's paramount duty is to the client, and where he finds that there is conflict of interests, he should refrain from doing anything, which would harm any interests of his client. A lawyer when entrusted with a brief is expected to follow the norms of professional ethics and try to protect the interests of his client in relation to whom he occupies a position of trust. The State Bar Council however found the appellant guilty of the second charge viz. that he had procured the brief of the complainant in another case on a fee of Rs. 300/- on the representation that he would secure a favourable report from the Radiologist showing that there was a fracture of the skull. The appellant was guilty of reprehensible conduct. The preamble to Chapter II Part VI of the Rules lays down that an advocate shall at all times, comfort himself in a manner befitting his status as an officer of the Court, privileged member of the community and a gentleman. R. 4 of this Chapter provides that an advocate shall use his best effort to restrain and prevent his client from resorting to sharp and unfair practices etc. There is a long catena of decisions laying down that offering of bribe or giving bribe or taking money from the client for the purpose of giving bribe amounts to grave professional misconduct.

    The lapse on the part of the appellant was perhaps due to the fact that in the struggle for existence he had to resort to such malpractices and thus to meet the ends of justice, he was suspended for a period of 1 year.

    Engagement In Business
    Conflict between interest and duty arises when an advocate is engaged in has taken full time service or part time service or engages in business or any avocation inconsistent with his practicing as an advocate.

    In the case of Bhupinder Kumar Sharma v. Bar Association, Pathankot , the appellant was guilty of professional misconduct as he was carrying on and continued his business and business activities even after his enrolment as an Advocate. He was running a photocopier documentation centre, PCO/STD booth in the court compound, Pathankot and he was proprietor/General Manager of the Punjab Coal Briquettes, Pathankot, a private concern and he was pursuing the business/his interest in the said business even on the date when his statement was recorded by the Disciplinary Committee. Thus it was in direct conflict with Rule 43 of Bar Council of India.

    Investment of savings by an advocate does not necessarily amount to engagement in money lending business and thus there is no conflict between interest and duty. On the other hand, if investments were made as a matter of regular business and for gain, it may constitute engagement in money-lending business as was held in the case of In Re Bhairo Dutt Bhandari, an Advocate.

    In the case of Madhav M. Bhokarikar v. Ganesh M. Bhokarikar (Dead) through Lrs. it was held that the appellant was an advocate duly enrolled under the provisions or the Advocates Act, 1961. Proceedings were initiated against the appellant by the Disciplinary Committee of the State Bar Council on a complaint made by the respondent accusing the appellant of having taken dealership of a retail outlet of petroleum products.

    On enquiry, the Disciplinary Committee of the State Bar Council found the charge proved. According to the State Bar Council, the appellant, though a practising advocate applied for the dealership and subsequently secured a letter of intent in his favour. Thereafter, the petrol pump was also started. The complaint was made after about two years of the retail outlet having remained in operation. During the pendency of the enquiry against the appellant, he entered into a partnership with his younger brother wherein the mutual arrangement arrived at between the two partners was that the appellant would remain a sleeping partner and his younger brother would actively and wholly look after the business.

    On 20.6.1998, the State Bar Council having held the appellant guilty of professional misconduct directed his license to practice to be suspended for a period of one year under Clause (c) of Sub-section (3) of Section 35 of the Act.

    Making Reckless And False Allegations
    A counsel is to assist the Court in the administration of justice and is not a mere mouthpiece of his client. If a counsel makes reckless and false allegations against a Magistrate in the application for transfer on the instructions of his client without taking any steps to verify the truth of these allegations, he is unfit to enjoy the privileges conferred upon him by law and must be visited with punishment as was held in the case of Ganwar v. Emperor. Thus a conflict is created between the Preamble (which imposes a duty on a lawyer) and the interest of the client and the lawyer.

    Where Lawyer Has A Personal Interest

    A fiduciary duty concerns disclosure of material facts in a situation where the fiduciary has either a personal interest in the matter to which the facts are material or acts for another party who has such an interest.

    The classic case where the duty arises is where a solicitor acts for a client in a matter in which he has a personal interest. In such a case there is an obligation on the solicitor to disclose his interest and, if he fails so to do, the transaction, however favorable it may be to the client, may be set aside at his instance as was held in the case of Lewis v Hillman.

    Throwing Away Interests
    Under the Legal Practitioners Act, Section 13 talked of throwing away interests of unimportant client in favour of important client is deserving of censure. In the case of In the matter of N, a pleader , a pleader N had two cases to attend on the same day. On the next day however N presented himself in the former case and the latter case which was weak was conducted by another pleader and the accused was convicted.

    The Court held that N may also have been influenced by the probability that in one case the client had very little chance of acquittal and that the case could not be made any worse by his absence and could not be made any better by his presence but however that may be the more difficult the case the more important …
    The court further held that in these circumstances he certainly on his own account of the proceedings threw away the interests of an unimportant client in favour of the interests of an important client and in so doing he has committed an offence which is deserving of the most serious censure. But a censure in a case of this kind is an inadequate punishment and we feel it our duty to mark our opinion of the gravity of the offence and also to protect the reputation of the profession itself. The reputation of the profession is liable to be gravely affected by conduct of this kind. We therefore order that N be suspended from practice for a period of two months from this date.

    Fees Not Paid
    In R. D. Saxena v. Balram Prasad Sharma, this Court has held that advocates have no lien over the papers of their client. It is held that, at the most the advocate may resort to legal remedies for, unpaid remuneration. It has been held that the right of the litigant to have the files returned to him is a corresponding counterpart of the professional duty of the advocate and that dispute regarding fees would be decided in an appropriate proceeding in Court.

    In the case of New India Assurance Co. Ltd. v. A.K. Saxena it was held that Respondent was an advocate on panel of the appellants - Some dispute arose between the appellants and the respondent as a result of which the respondent was asked to return all papers - The respondent was willing to return the papers provided that all his fees were paid - Advocates have no lien over the papers of their clients and at the most the advocate may resort to legal remedies for, unpaid remuneration - The right of the litigant to have the files returned to him is a corresponding counterpart of the professional duty of the advocate and that dispute regarding fees would for fees is to be decided in an appropriate proceeding in Court.

    Not To Be Bias

    In the case of Satyendra Narain Singh and Ors. v. Ram Nath Singh and Ors. It was held that there are a few black sheep in every profession, nay, in every walk of life. But few as they are, they tarnish, by their machinations, the fair name of age-old institutions. Therefore, persons who occupy high public offices must take care to see that those who claim to be close to them are not allowed to exploit that closeness, alleged or real. On the facts of this case, we will only say that Shri Sailendra Kumar Jha took a correct decision in not appearing in the case any further and, with respect, his father Justice S.K. Jha acted in the best traditions of the Judiciary in seeing that his son withdrew from the case. It is better that in such circumstances the advocate son, rather than the Judge father, withdraws from the case.

    De Grandpré, J. in his dissent in Committee for Justice and Liberty et al. v. National Energy Bd. , at pp. 394-395:
    [...] the apprehension of bias must be a reasonable one, held by reasonable and right-minded persons, applying themselves to the question and obtaining thereon the required information. In the words of the Court of Appeal [at p 667], that test is "what would an informed person, viewing the matter realistically and practically -- and having thought the matter through -- conclude.

    This approach was subsequently reiterated by the Supreme Court in R. v. R.D.S. , and Wewaykum Indian Band v. Canada , and followed in numerous lower court decisions including, most recently, the Federal Court of Appeal in Canada (Attorney General) v. Fetherston

    The law in England is also the same and has been stated in the Halsbury's Laws of England.

    If counsel who has advised on or been engaged in a case is raised in the Bench, and the same case comes before him, the practice is for him to refuse to adjudicate on it.
    In Manak Lal v. Prem Chand, a complaint alleging professional misconduct against Manak Lal, an advocate of the Rajasthan High Court, was filed by Prem Chand. The bar council tribunal, appointed by Chief Justice of the High Court to enquire in to the alleged misconduct of Manik Lal, consisted of the Chairman and two other members. The Chairman had earlier represented Prem Chand in a case. He was, however, a senior advocate and was once advocate general of the Rajasthan High Court. The Supreme Court had no hesitation in assuming that the Chairman had no personal contact with this client and he did not remember that he had appeared on his behalf in certain proceedings.

    The court was thus satisfied that there was no real likely hood bias, but still it held that the Chairman was disqualified on the ground justice not only be done but must appear to be done to the litigating public.

    Conflicts of Interest Between Claimholders, Lawyers And Litigation Entrepreneurs

    All agreements that obstruct or affect administration of justice would be treated as invalid under Section 23 of the Indian Contract Act, 1872. The agreement between a lawyer and his client is directly concerned with the administration of justice. It is an agreement between a lawyer, who is an officer of the Court and who is given privilege of audience by the Court, and his client, who is a suitor in the Court and has a cause to be tried by the Court. If such an agreement tends either directly or indirectly to affect the administration of justice or sully its course it would always be declared to be invalid.

    Detachment and objectivity, which are the basis of the strength of the Bar, cannot be retained when a lawyer agrees to share in the profits of litigation. An agreement between lawyer and client making the payment of the lawyer's fees conditional upon the success of the suit and giving the lawyer an interest in the subject matter of the suit itself, would necessarily tend to undermine the status of lawyer as a lawyer. Such an agreement has always been condemned as unworthy of the legal profession and is opposed to a fundamental rule of professional ethics as was held in the case of In Re, K. L. Gauba.

    An undertaking on the part of the practitioner to bear expenses of litigation on the promise of the litigant that a certain portion of the net profits of the litigation will be allowed to the former in case of success is grossly improper under this section. It is grossly improper professional duty on the part of a practitioner to enter into an a agreement with his client to give him money and legal aid in consideration of his assigning over to him a part of the property, the subject matter of the suit, in the event of success.

    The same view was expressed by a full bench of the Calcutta High Court in In the matter of an advocate. Chief Justice Maclean, who delivered the principal judgment of the full bench, observed that it is professional misconduct for an advocate to agree with his client to accept as his fee for share of the property, fund or other matter in the litigation for his services as an Advocate in such litigation upon the successful issue thereof.

    Suggestion
    Bar Council of India Rules is devoid of any specific provision regarding Conflict between interest and duty. As I have made detailed analysis of the position regarding conflict in America, England, Canada, Philippines and the thought provoking that comes out is that why is such a provision lacking in India. In Canada, the rules even incorporate guiding principles as commentaries but as far as India is concerned the word conflict has no place in the rules. Thus considering the present scenario and complexity of the legal profession, it is recommended that there be a specific provision regarding Conflict between Interests and Duties of a Lawyer.

    Conclusion
    Where a lawyer is guilty of a conflict of interest in representing a client he will have committed a breach of duty. That duty is usually expressed as a fiduciary obligation arising out of the relationship between solicitor and client. But there is similar duty owed by the lawyer to the court (as well as an ethical duty). The duty to the court arises from the court's concern that it should have the assistance of independent legal representation for the litigating parties. The integrity of the adversarial system is dependant on lawyers acting with perfect good faith. This is central to the preservation of public confidence in the admission of justice.
    The usual basis for restraining a lawyer from acting for a client on the ground of conflict of interest is that a conflict is perceived between the continuing duty of the lawyer (owed to his former client) not to disclose or use the latter's prejudice that which he learned confidentiality, and the interest he has in advancing the case of his new client.

    Every counsel has a duty to his client fearlessly to raise every Issue, advance every argument and ask every question, however distasteful, which he thinks will help his client's case. As an officer of the Court concerned in the administration of justice, he has an overriding duty to the Court to the standards of his profession, and to the public which may and often does lead to a conflict with his client's wishes or with what the client thinks are his personal interests.
    Counsel must not mislead the Court, he must not lend himself to casting aspersions on the other party or witnesses for which there is no sufficient basis in the information in his possession, he must not without authorities or documents which may tell against the clients but which the law or the standards of his profession require him to produce. By so acting he may well incur the displeasure or worse of his client so that if the case is lost, his client would or might seek legal redress if that were open to him.

    In an era of national firms and a rising turnover of lawyers, especially at the less senior levels, the imposition of exaggerated and unnecessary client loyalty demands, spread across many offices and lawyers who in fact have no knowledge whatsoever of the client or its particular affairs, may promote form at the expense of substance, and tactical advantage instead of legitimate protection. Lawyers are the servants of the system, however, and to the extent their mobility is inhibited by sensible and necessary rules imposed for client protection, it is a price paid for professionalism. Business development strategies have to adapt to legal principles rather than the other way around. Yet it is important to link the duty of loyalty to the policies it is intended to further. An unnecessary expansion of the duty may be as inimical to the proper functioning of the legal system as would its attenuation. The issue always is to determine what rules are sensible and necessary and how best to achieve an appropriate balance among the competing and conflicting interests. He is not merely a mouthpiece of his client to say what he wants. He must disregard to most specific instructions of his client, if they conflict with his duty to the Court.

    No better words can sum up this project then that of Gandhi Ji. He said the following: -
    … that the duty of a lawyer was to place correct facts before the judge and to help him to arrive at the truth, and not to prove the guilty as innocent.

    Bibliography
    # Lawyer's Duty to the Court - (1998) 114 L.Q.R 93
    # Legal Doubletalk and the Concern with Positional Conflicts: A "Foolish Consistency"? - 111 Penn St. L. Rev. 1
    # Conflicts of Interests between Claimholders, Lawyers and Litigation Entrepreneurs - (2007) 19.1 Bond Law Review.
    # Unforeseeable Concurrent Client Conflicts: The Committee on Professional and Judicial Ethics - 61 The Record 102 (2006)
    # Conflicts of Duty : The Perennial Lawyer's Tale - A comparative study of the Law in England and Australia - 30 Melbourne U.L.R. 88
    # A Tribute to the Honorable Wallace P. Carson, Jr.: Lawyer-Client Conflicts of Interest Law: Contributions of Chief Justice Wallace P. Carson, Jr. During a time of Dynamic change - 43 Williamette Law Review 527.
    # Principles of Administrative Law - M.P. Jain
    # Solutions to the City Attorney's Charter - Imposed Conflict of Interest Problem - 66 Ohio State Law Journal 1075.
    # Advocates Practice (ILI).
    # Creating Conflicts of Interest: Litigation as Interference with the Attorney-Client Relationship - 43 American Business Law Journal 173.
    # P Ramanatha Aiyer; Legal and Professional Ethics; 3rd edition; 2003.
    # All India Reporter
    # All England Reporter
    # Canadian Court Cases

    About The Author:
    Amanjot Malhi - I am a law graduate and have just completed my graduation in LLB (H) from University School of Law and Legal Studies, Guru Gobind Singh Indraprastha University, Kashmere Gate, Delhi. Presently, I am doing internship at a law firm.

    Print This Article

    How To Submit Your Article:

    Follow the Procedure Below To Submit Your Articles

    Submit your Article by using our online form Click here
    Note* we only accept Original Articles, we will not accept Articles Already Published in other websites.
    For Further Details Contact: [email protected]


    Divorce by Mutual Consent in Delhi/NCR

    Mutual DivorceRight Away Call us at Ph no: 9650499965

    File Your Copyright - Right Now!

    Copyright Registration
    Online Copyright Registration in India
    Call us at: 9891244487 / or email at: [email protected]